letsthinkcritically
letsthinkcritically
  • 327
  • 9 439 827

Video

Equation on Sum of Powers
zhlédnutí 8KPřed rokem
Equation on Sum of Powers
Multiples of Primes | Irish National Mathematical Olympiad 2007
zhlédnutí 6KPřed rokem
Multiples of Primes | Irish National Mathematical Olympiad 2007
Solving This Equation With One Simple Trick
zhlédnutí 9KPřed rokem
Solving This Equation With One Simple Trick
Sum of Prime Squares is a Cube
zhlédnutí 7KPřed rokem
Sum of Prime Squares is a Cube
Solving This Problem With One Simple Trick
zhlédnutí 8KPřed rokem
Solving This Problem With One Simple Trick
When is This Fraction an Integer?
zhlédnutí 9KPřed rokem
When is This Fraction an Integer?
Equation of Powers Solved With One Simple Trick | Japan MO Finals
zhlédnutí 8KPřed rokem
Equation of Powers Solved With One Simple Trick | Japan MO Finals
Solving This Equation With One Simple Trick | Baltic Way 1992
zhlédnutí 6KPřed rokem
Solving This Equation With One Simple Trick | Baltic Way 1992
A Tricky Divisibility Problem
zhlédnutí 9KPřed rokem
A Tricky Divisibility Problem
Solving This Problem in One Step | Baltic Way 2011
zhlédnutí 34KPřed rokem
Solving This Problem in One Step | Baltic Way 2011
Equation on Symmetric Polynomials | Balkan MO 2017
zhlédnutí 9KPřed rokem
Equation on Symmetric Polynomials | Balkan MO 2017
When is p^2-p+1 a Cube? | Balkan MO 2005
zhlédnutí 13KPřed rokem
When is p^2-p 1 a Cube? | Balkan MO 2005
Are There Any Fifth Powers?
zhlédnutí 6KPřed rokem
Are There Any Fifth Powers?
An Interesting Equation of Powers
zhlédnutí 8KPřed rokem
An Interesting Equation of Powers
A Beautiful Equation | Switzerland IMO TST 2015
zhlédnutí 9KPřed rokem
A Beautiful Equation | Switzerland IMO TST 2015
A Quick System of Sum of Powers | Norwegian Abel Maths Competition
zhlédnutí 7KPřed rokem
A Quick System of Sum of Powers | Norwegian Abel Maths Competition
When is This Fraction a Cube?
zhlédnutí 11KPřed rokem
When is This Fraction a Cube?
A Nice Equation of Powers | Turkish National Maths Olympiad 2014
zhlédnutí 7KPřed rokem
A Nice Equation of Powers | Turkish National Maths Olympiad 2014
When the Sum of Powers are All Equal
zhlédnutí 8KPřed rokem
When the Sum of Powers are All Equal
2 Equations 3 Unknowns | Turkish Junior Mathematics Olympiad 2021
zhlédnutí 6KPřed rokem
2 Equations 3 Unknowns | Turkish Junior Mathematics Olympiad 2021
A Quick Functional Equation | India IMO TST 2010
zhlédnutí 10KPřed rokem
A Quick Functional Equation | India IMO TST 2010
Parabolas Solved in Two Ways
zhlédnutí 5KPřed rokem
Parabolas Solved in Two Ways
Indian National Mathematics Olympiad 2013 Problem 2
zhlédnutí 6KPřed rokem
Indian National Mathematics Olympiad 2013 Problem 2
A Quick Problem of Ratios
zhlédnutí 8KPřed rokem
A Quick Problem of Ratios
The Answer is Surprisingly Easy! | India National Mathematics Olympiad 2003
zhlédnutí 18KPřed rokem
The Answer is Surprisingly Easy! | India National Mathematics Olympiad 2003
One Simple Trick to Solve This National Maths Olympiad Problem
zhlédnutí 9KPřed rokem
One Simple Trick to Solve This National Maths Olympiad Problem
Could You Make The Greece IMO Team? | Greece IMO TST 2013
zhlédnutí 7KPřed rokem
Could You Make The Greece IMO Team? | Greece IMO TST 2013
When Can This be a Perfect Square? | Turkish National Mathematical Olympiad 2009
zhlédnutí 13KPřed rokem
When Can This be a Perfect Square? | Turkish National Mathematical Olympiad 2009
Turkish National Maths Olympiad 2004
zhlédnutí 7KPřed rokem
Turkish National Maths Olympiad 2004

Komentáře

  • @dkr1998
    @dkr1998 Před 14 hodinami

    nice

  • @krstev29
    @krstev29 Před 22 hodinami

    This is a little bit hard for a 1987 P4, but I have a different approach than this

  • @narayansareekuthir5330

    No sol

  • @vbcool83
    @vbcool83 Před dnem

    Nice problem!

  • @user-hk3tl1fn6u
    @user-hk3tl1fn6u Před 2 dny

    K I liked your solution very much

  • @Mehmet42832
    @Mehmet42832 Před 3 dny

    The Pythagorean triple part was very well thought out.

  • @br75857
    @br75857 Před 4 dny

    Bro change the background screen black. It will be good for our eyes

  • @franciscook5819
    @franciscook5819 Před 4 dny

    2009=7²41 √a=√(2009) - √b a=2009+b-2√(2009b) but a is an integer so √(2009b) = 7√(41b) is an integer so b=y²41, similarly (symmetry) a=x²41 original eqn => x√41 + y√41 =7√41 x + y = 7 so x runs from 0 to 1 as y runs from 7 to zero thus solutions (of the form n²41) are ... 0,2009 41,1476 164,1025 369,656 656,369 1025,164 1476,41 2009,0

  • @user-hk3tl1fn6u
    @user-hk3tl1fn6u Před 4 dny

    Why not provide proof problems ?

  • @NeerajKumar-gk9kz
    @NeerajKumar-gk9kz Před 4 dny

    Hi can u tell me how approxh to solve putnam problam

  • @YTZMath
    @YTZMath Před 5 dny

    I have a simpler solution: assume such f(x) exist if f(x) < x + 1987 / 2 f(f(x)) < f(x) + 1987 / 2 < (x + 1987 / 2) + 1987 / 2 = x + 1987, conflict with f(f(x)) = x + 1987 so f(x) >= x + 1987 / 2 , this is conclusion 1 if f(x) > x + 1987 / 2 f(f(x)) > f(x) + 1987 / 2 > (x + 1987 / 2) + 1987 / 2 = x + 1987, conflict with f(f(x)) = x + 1987 so f(x) <= x + 1987 / 2 , this is conclusion 2 combine conclusion 1 and 2: f(x) = x + 1987 / 2 f(0) = 1987 /2 is not integer, conflict with f(x) belong to Z so such f(x) not exist

  • @franciscook5819
    @franciscook5819 Před 5 dny

    I used your second method - knowing that powers of x+1/x to the 2ⁿ "look nice"! Your final explanation should have mentioned that if the two "powers" are adjacent (e.g. 3 and 4 or 11 and 12) you get an x+1/x term in the middle. Which is perhaps easier than dealing with (e.g. 2 and 5) x³+1/x³ in the middle.

  • @franciscook5819
    @franciscook5819 Před 7 dny

    I did it differently. A combination of my version after your "power of ten" extraction would have been quickest. (Find the 1000100010001 then long divide). GCF: divide greater by smaller until remainder = 0 (so old remainder becomes new divisor) the last divisor is the GCF Clearly a factor of 1000 can be taken out of the denominator 𝟸𝟸𝟺𝟺𝟾𝟻𝟷𝟺𝟾𝟻𝟷𝟺𝟾𝟻𝟷𝟺𝟼𝟸𝟽 𝟾𝟷𝟷𝟾𝟾𝟷𝟷𝟾𝟾𝟷𝟷𝟾𝟾𝟷𝟷𝟾 ×𝟸𝟶𝟶 𝟷𝟼𝟸𝟹𝟽𝟼𝟸𝟹𝟽𝟼𝟸𝟹𝟽𝟼𝟸𝟹𝟼𝟶𝟶 - 𝟼𝟸𝟷𝟶𝟾𝟿𝟷𝟶𝟾𝟿𝟷𝟶𝟾𝟿𝟷𝟶𝟸𝟽 𝟾𝟷𝟷𝟾𝟾𝟷𝟷𝟾𝟾𝟷𝟷𝟾𝟾𝟷𝟷𝟾 ×𝟽𝟶 𝟻𝟼𝟾𝟹𝟷𝟼𝟾𝟹𝟷𝟼𝟾𝟹𝟷𝟼𝟾𝟸𝟼𝟶 - 𝟻𝟸𝟽𝟽𝟸𝟸𝟽𝟽𝟸𝟸𝟽𝟽𝟸𝟸𝟽𝟼𝟽 𝟾𝟷𝟷𝟾𝟾𝟷𝟷𝟾𝟾𝟷𝟷𝟾𝟾𝟷𝟷𝟾 ×𝟼 𝟺𝟾𝟽𝟷𝟸𝟾𝟽𝟷𝟸𝟾𝟽𝟷𝟸𝟾𝟽𝟶𝟾 - 𝟺𝟶𝟻𝟿𝟺𝟶𝟻𝟿𝟺𝟶𝟻𝟿𝟺𝟶𝟻𝟿 𝟾𝟷𝟷𝟾𝟾𝟷𝟷𝟾𝟾𝟷𝟷𝟾𝟾𝟷𝟷𝟾 𝟺𝟶𝟻𝟿𝟺𝟶𝟻𝟿𝟺𝟶𝟻𝟿𝟺𝟶𝟻𝟿 𝚡𝟸 𝟾𝟷𝟷𝟾𝟾𝟷𝟷𝟾𝟾𝟷𝟷𝟾𝟾𝟷𝟷𝟾 - 0 𝚐𝚌𝚏 = 𝟺𝟶𝟻𝟿𝟺𝟶𝟻𝟿𝟺𝟶𝟻𝟿𝟺𝟶𝟻𝟿 𝚜𝚘 𝚌𝚊𝚗𝚌𝚎𝚕 𝚝𝚘𝚙 𝚊𝚗𝚍 𝚋𝚘𝚝𝚝𝚘𝚖 denominator = 2𝚡1000 numerator : 𝟸𝟸𝟺𝟺𝟾𝟻𝟷𝟺𝟾𝟻𝟷𝟺𝟾𝟻𝟷𝟺𝟼𝟸𝟽 𝟺𝟶𝟻𝟿𝟺𝟶𝟻𝟿𝟺𝟶𝟻𝟿𝟺𝟶𝟻𝟿 𝚡𝟻𝟶𝟶 𝟸𝟶𝟸𝟿𝟽𝟶𝟸𝟿𝟽𝟶𝟸𝟿𝟽𝟶𝟸𝟿𝟻𝟶𝟶 - 𝟸𝟷𝟻𝟷𝟺𝟾𝟻𝟷𝟺𝟾𝟻𝟷𝟺𝟾𝟻𝟷𝟸𝟽 𝟺𝟶𝟻𝟿𝟺𝟶𝟻𝟿𝟺𝟶𝟻𝟿𝟺𝟶𝟻𝟿 𝚡𝟻𝟶 𝟸𝟶𝟸𝟿𝟽𝟶𝟸𝟿𝟽𝟶𝟸𝟿𝟽𝟶𝟸𝟿𝟻𝟶 - 𝟷𝟸𝟷𝟽𝟾𝟸𝟷𝟽𝟾𝟸𝟷𝟽𝟾𝟸𝟷𝟽𝟽 𝟺𝟶𝟻𝟿𝟺𝟶𝟻𝟿𝟺𝟶𝟻𝟿𝟺𝟶𝟻𝟿 𝚡𝟹 𝟷𝟸𝟷𝟽𝟾𝟸𝟷𝟽𝟾𝟸𝟷𝟽𝟾𝟸𝟷𝟽𝟽 - 0 𝟻𝟻𝟹/𝟸𝟶𝟶𝟶

  • @franciscook5819
    @franciscook5819 Před 7 dny

    No wonder this was double starred and put last on the test. Unless you are aware of "Vieta jumping" the chances of finding the solution in a short time are small. There's even a Wikipedia entry which solve this.

  • @franciscook5819
    @franciscook5819 Před 7 dny

    Much the same but slightly differently ... natural numbers s.t. 1<a<b<c and P=(a-1)(b-1)(c-1) | abc-1 or k(a-1)(b-1)(c-1) = abc-1 ...(1) ...(1) => k<abc/( (a-1)(b-1)(c-1) ) ...(2) note that for any natural number n>1, we have n/(n-1)>(n+1)/n (because n²>n²-1) and a≥2,b≥3,c≥4 so max value of ...(2) is for min, a,b,c from ...(2) k<24/(6)=4 k≤3 k=1 => ac+bc+ab-a-b-c=0 but ab > a and bc > b and ac > c, so k≠1 looking at a, if a≥4 k=(a/(a-1))(b/(b-1))(c/(c-1))-1/P ≤ (4/3)(5/4)(6/5)-1/P < 2 so no solution for a≥4 if a=3 k≤(3/2)(4/3)(5/4)-1/P < 2.5 so k=2 first consider a=2 k≤(2/1)(3/2)(4/3)-1/P < 4 so k=2 or 3 if k=3, 3(2-1)(b-1)(c-1)=2bc-1 3bc-2bc-3b-3c+3+1=0 bc-3b-3c+4=0 add 5 and factorise (b-3)(c-3)=5 factor pairs 1,5 solutions a,b,c = 2,4,8 if k=2 2(2-1)(b-1)(c-1)=2bc-1 -2b-2c+3=0 2b+2c=3 has no solutions for b,c > a=2 consider a=3 and k=2 2(3-1)(b-1)(c-1)=3bc-1 4bc-4b-4c+4-3bc+1=0 add 11 and factorise (b-4)(c-4)=11 factor pairs 1,11 solutions a,b,c = 3,5,15

  • @scottychen2397
    @scottychen2397 Před 8 dny

    This is a beautiful problem: At the solution, Can you say that actually, a = 1. … and then it’s a little suspect what youre doing with this. Since one is summing over integers strictly not on the prime ideal, The denominator isn’t carrying factors of this prime. So youre looking at a rational: something in lowest form 1979/… … by just observing its definition: No spooky logic implied, Doesn’t have any factors of this prime. … which makes me think its not too good of a problem after all these years.. I know thats not what theyre asking. It’s interesting to see if you can find this symetry elsewhere in this summation to find out if one can write this as 1979*a where a isn’t actually 1: This would beget spooky logic. If 1979 isn’t actually a prime: then none of this holds … - but one can’t be unsure about this pre-argumentation The only thing one can call beautiful is: The sameness between (1) Taking away a subsequence, additively (2) giving that same sequence, additively, whilst taking away .. the ring element ‘2’ multiplied by the same thing one is giving. This is the kind of trick one is using here.

  • @supalupallama
    @supalupallama Před 8 dny

    I love the content

  • @iannalemme
    @iannalemme Před 8 dny

    this is brilliant

  • @rooster5572
    @rooster5572 Před 9 dny

    can anyone explain how he got the numerator to be 1979 in the summation?

  • @FluffyPhoenix991
    @FluffyPhoenix991 Před 9 dny

    Eh am I stewpid or shouldn't it be 1+½+...+1/1319 - ½(1+½+...+1/658) instead of -2(...). Also there are so many mistakes (as you've noticed) that make this simple step redundantly hard to understand.

  • @Aurora-ux9vb
    @Aurora-ux9vb Před 9 dny

    ummm, a much easier approach, or the most "visible" one; simply (x+1/x)³-3(x+1/x)=x³+1/x³ or 216-3(6)=198..... thats it

  • @saswatipal2504
    @saswatipal2504 Před 10 dny

    The more difficult version is the set of such sequence of 1's is never a perfect prime power

  • @xaxi222
    @xaxi222 Před 11 dny

    If n-m must be multiple of 100 (i.e. n-m≥100) and also m≥1, doesn't that mean that min(n+m) is for n=101 and m=1, thus min(n+m)=102? And obviously 101=n≥3, so that also checks out in this case, so isn't the answer 102 instead of 106 or am I missing something?

  • @user-tj1yr7gs5e
    @user-tj1yr7gs5e Před 14 dny

    Лемма Титу.

  • @lifeisacompetation
    @lifeisacompetation Před 14 dny

    Keep in mind "an Asian" Is solving this!

  • @xaxi222
    @xaxi222 Před 15 dny

    Given the condition that y≥4 at 2:43 then the corresponding solution at 3:30 should indeed be (y,z)=(2t,3t), where for t∈ℕ, but only for t>1 (for t=1 => y=2<4). But one might then think "but (x,y,z)=(1,2,3) is obviously a solution". Yeah, but it's a result from the solution found at 6:46, not the one at 3:30

  • @BesnikDule-fl4yb
    @BesnikDule-fl4yb Před 15 dny

    I think was a very easy imo problem.

  • @xaxi222
    @xaxi222 Před 15 dny

    The expansion of (a(x+1)+b)(cx+d) at 5:30 should have a bcx term instead of the bdx term shown in the video

  • @calculus-is-fun
    @calculus-is-fun Před 17 dny

    do i have any problem for this solution? since proven f is 1-1 and f(x+1987)=f(x)+1987, then f(x+1987)-f(x)=1987, hence f is a linear function (since the difference between every set of (x,x+1987) must be a constant), which x>0. Then, since f(f(x))=x+1987, m(mx+b)+b=x+1987, which we can get m=1,-1(rejected) and b=1987/2. considering that previously we know x>0 and f defined in Z>0, then looking back to the f(x)=x+1987/2, which firstly f is not integer for integer inputs, and it is also not defined in [0,1987/2). Therefore, we can claim that no such function.

  • @lechaiku
    @lechaiku Před 18 dny

    I've created 2 years ago a shortcut for such math problems: √[n (n+1) (n+2) (n+3) + 1] ------> for 4 consecutive numbers + 1 5n + (n -1 )^2 --------------------- 5 * 1000 + 1000000 - 2000 + 1 = 1000000 + 3001 = 1003001

  • @graham741
    @graham741 Před 19 dny

    LHS must be even assuming it's not an even prime (but p=2 works too) so then RHS is odd, meaning it's congruent to 0 mod 4. Therefore p^3 + 1 = 0 mod 4 so p must be 3 mod 4, and primes must be 1 or 5 mod 6, so then p must be 11 or 7 mod 12. The only primes generated by this are 7,11,19,23,31, and 43 and checking yields p = 7,11 (it's annoying if you don't have a calculator but very doable with some mod checks + divisibility checks, eg if it's congruent to 0 modulo by n and not n^2 where n is not a perfect square then it cannot be a perfect square)

  • @mathsandillathensclasses634

    I've the easiest and shortest method. It can be solved in a minute or two.

  • @Krish_202
    @Krish_202 Před 22 dny

    How about using weirstrass inquealitiy considering a2a3......an=1 but its not given that a2a3........an<1

  • @niom9446
    @niom9446 Před 27 dny

    bro is insane

  • @Sherif-Academy
    @Sherif-Academy Před 28 dny

    czcams.com/video/7-GkLcIZqiQ/video.html outstanding!!!

  • @minhphamnhat7991
    @minhphamnhat7991 Před 29 dny

    Where is 1/659?

  • @karolkurek9201
    @karolkurek9201 Před měsícem

    I didn't grab a finer point: how you went from 2^(x-1) congruent to 0 mod 8 to y+1+2^(x-1)=7*2^(2x-3). Can someone explain why k must be 2x-3, not 2x-1 or 2x+7 and so on?

  • @lechaiku
    @lechaiku Před měsícem

    Another approach without using modulo and log (this is "no-pen-no-paper" method): 1. the last digit of x must be 9, because only 9 goes in cycles (9,1) with the 13th power being 9. 2. 100^13 is 27-digit number, so x must be 2-digit number ----> 26(digits) : 13 = 2. 3. x can't be 99, because the given 26-digit number is too small (it's obvious). 4. 8 = (2^3)^13 = 2^39; let's calculate 2^40 5. 2^10 = 1024, let's use "1000" 6. 2^40 = approx. 8^13 = approx 1000 ^4 7. 80^13 = approx 1000 ^4 * 10^13 = 10^12 * 10^13 = 10^25 8. It is less than the given 26-digit number 9. therefore x = 89

  • @drynshockgameplays
    @drynshockgameplays Před měsícem

    How would you choose the squares if you got something like m^4 + 3m^3 + ... with the cubic term

  • @eloaba57
    @eloaba57 Před měsícem

    I got lost on the sumation part (note im finishing calc 1 so xd)

  • @MrEliseoD
    @MrEliseoD Před měsícem

    Two cool facts about 13th powers is that a) the answer is approximately 13 times the length of the number; and b) that the last digit is always the last digit of the number we’re after… Given this, we can confidently say this is a two digit number ending with 9 (26 digits long, very convenient!), given 100^13 has 27 digits! The digit sum is not divisible by 3 or 9, so that leaves 39, 69 and 99 out… Another trick to know is that for 13th powers the second to last digit (which we can call x) can be used to determine the second to last digit of the root… in the case of a digit ending with 9, the value of the root’s second to last (technically first) digit would be 7(x-2) mod 10… equal to 7 x (6-2) mod 10 = 8… that would mean in our case that the 13th root of our number is 89…

  • @michaelg3490
    @michaelg3490 Před měsícem

    Can someone help me with where 2p<=q+6 comes from? Thank you!

  • @user-zf4si4go7t
    @user-zf4si4go7t Před měsícem

    Vieta jumping and we done

  • @sproutssupport7789
    @sproutssupport7789 Před měsícem

    And why did you add u? When you can easly 1²-2×1×√2+1+(√2+1)²

  • @pichamonnaksomboon
    @pichamonnaksomboon Před měsícem

    ทำไปแล้ว

  • @user-fd5ey5ch5c
    @user-fd5ey5ch5c Před měsícem

    Hk?

  • @marcogallardo-my7hb
    @marcogallardo-my7hb Před měsícem

    (3,4,5) and (3,5,6) and (4,3,5) and ( (4,5,3) and (5,3,4) and (5,4,3) is Solution. => n! Is Solution for n= 3, 3!=6 => For one Solution => 6 add. Total 30 solutions combined

  • @michaelaristidou2605
    @michaelaristidou2605 Před měsícem

    Couldn't A1 = B ? Because if it could, then we don't have a contradiction.

  • @Andrei-sw3my
    @Andrei-sw3my Před měsícem

    My solution : If you subtract 1 from a multiple of seven , that number will either be odd or factor into an odd * even . As 2^n is always even * even we can never have this relation .

    • @xinpingdonohoe3978
      @xinpingdonohoe3978 Před měsícem

      You'll have to prove that it might factor into odd×even. You don't have to prove it could be odd, because it will obviously either be odd or even, but you must show that this 7k-1 always has an odd factor.

    • @Andrei-sw3my
      @Andrei-sw3my Před měsícem

      @@xinpingdonohoe3978 to prove just take it by cases : 1. K is even then k = 2m, we have 7(2m)-1 = 14m-1=odd. Case 2. K is odd then k= 2m + 1 , we have 7(2m+1)-1 = 14m - 6 = 2(7m-3) = even * odd . And as 2^n Is always even * even we cannot have this relation .

  • @Andrei-sw3my
    @Andrei-sw3my Před měsícem

    My solution : If you subtract 1 from a multiple of seven , that number will either be odd or factor into an odd * even . As 2^n is always even * even we can never have this relation .